Can a Sequence of Elements in a Nonempty Subset Converge to Its Supremum?

  • Thread starter STEMucator
  • Start date
  • Tags
    Homework
In summary: Have fun! Let ##A## be a nonempty subset of ##ℝ## which is bounded above.I'm having a bit of trouble actually producing the sequence. I understand why I cannot pre-define A (I did it mostly to get an idea of what's going on ), but can I choose ANY convergent sequence or is there a method to doing this?The intention of my lemma is that it would define a nice sequence ##(x_n)_n##. The sequence as defined in the lemma can be shown to converge to ##sup(A)##.
  • #1
STEMucator
Homework Helper
2,076
140

Homework Statement



http://gyazo.com/27a104e05c409c75611ee4250a89c790

Homework Equations



Sup/Inf axioms as well as the ε-N definition.

The Attempt at a Solution



Suppose A is a nonempty subset of ℝ bounded above by ##M##. We want to show that ##lim_{n→∞} x_n = sup(A)## where ##x_n## is a sequence of elements of A.

That is, ##\forall ε > 0, \exists N \space | \space n ≥ N \Rightarrow |x_n - sup(A)| < ε##

Note that since A is bounded above by M, we know that ##M > a, \space \forall a \in A## including ##sup(A)##.

So :

##|x_n - sup(A)| ≤ |x_n| + |sup(A)| < |x_n| + M##

So ##|x_n|## is bounded above by ##ε - M## and below by ##-(ε+M)##

I'm not quite sure how to continue here.
 
Physics news on Phys.org
  • #2
You need to define the sequence ##(x_n)_n## first. Of course not every sequence will do.
 
  • #3
micromass said:
You need to define the sequence ##(x_n)_n## first. Of course not every sequence will do.

Ohhh I thought this was a general question, so I need a concrete sequence then?

How about ##A = \{2 - \frac{1}{n} \space | \space n = 1,2,... \}## and ##x_n = 2 - \frac{1}{n}##.

Then ##sup(A) = 2## and ##x_n → 2## as ##n→∞##

##|x_n - sup(A)| = |2 - \frac{1}{n} - 2| = \frac{1}{n}##

So choosing ##n ≥ \frac{1}{ε} \Rightarrow |x_n - sup(A)| < ε##

Didn't even need the upper bound except to establish sup(A) existed :).
 
  • #4
You'll need a concrete sequence. But the set ##A## is completely general. You can't pick ##A=\{2-1/n~\vert~n\in \mathbb{N}\}##. It needs to work for every possible set ##A## (that is bounded above).

So for a set ##A## that is bounded above, you need to find a concrete sequence that converges to ##sup(A)##.
 
  • #5
micromass said:
You'll need a concrete sequence. But the set ##A## is completely general. You can't pick ##A=\{2-1/n~\vert~n\in \mathbb{N}\}##. It needs to work for every possible set ##A## (that is bounded above).

So for a set ##A## that is bounded above, you need to find a concrete sequence that converges to ##sup(A)##.

I'm having a bit of trouble actually producing the sequence. I understand why I cannot pre-define A ( I did it mostly to get an idea of what's going on ), but can I choose ANY convergent sequence or is there a method to doing this?

I mean I can come up with enough convergent sequences like ##x_n = 1/n##, but it's not like I can put a finite number on sup(A).

All i really have is the fact that A is bounded above by M so that sup(A) exists.
 
  • #6
Here's a hint. Prove the following "lemma":

Given a positive integer ##n##. There is always an element ##x_n## such that ##sup(A)-\frac{1}{n}< x_n## and such that ##x_n\in A##.
 
  • #7
micromass said:
Here's a hint. Prove the following "lemma":

Given a positive integer ##n##. There is always an element ##x_n## such that ##sup(A)-\frac{1}{n}< x_n## and such that ##x_n\in A##.

I don't see how that helps me? I've been staring at it for awhile and I'm unclear about your intentions.

sup(A) < xn + 1/n

Then i was thinking induction.
 
  • #8
Zondrina said:
I don't see how that helps me? I've been staring at it for awhile and I'm unclear about your intentions.

sup(A) < xn + 1/n

Then i was thinking induction.

Induction won't help here.

The intention of my lemma is that it would define a nice sequence ##(x_n)_n##. The sequence as defined in the lemma can be shown to converge to ##sup(A)##.
 
  • #9
micromass said:
Induction won't help here.

The intention of my lemma is that it would define a nice sequence ##(x_n)_n##. The sequence as defined in the lemma can be shown to converge to ##sup(A)##.

Waaaait I see what you're getting at now. Since ##x_n + 1/n## will be a sequence as well.

I unfortunately just got called into work 2 mins ago, I will continue this later today when I get home.
 
  • #10
Zondrina said:
Waaaait I see what you're getting at now. Since ##x_n + 1/n## will be a sequence as well.

It will be a sequence as well, but I'm not sure how it helps...

I unfortunately just got called into work 2 mins ago, I will continue this later today when I get home.

Have fun!
 
  • #11
Let ##A## be a nonempty subset of ##ℝ## which is bounded above.

We must show ##\exists x_n \in A \space | \space \lim_{n→∞} x_n = sup(A)##

We must somehow construct ##x_n## from the elements of A so that it converges to sup(A).

Here's a hint. Prove the following "lemma":

Given a positive integer n. There is always an element ##x_n## such that ##sup(A)−1/n<x_n## and such that ##x_n∈A##.

Suppose n is a positive integer and ##x_n## is a sequence of elements of A. Since ##x_n \in A##, we know that ##x_n ≤ sup(A)## so that ##x_n## is bounded.

I think I have to use the fact it's bounded somehow, I'm having some thoughts, but I'd rather hear some input before going off in potentially the wrong direction.
 
  • #12
Sorry for the double, but I just woke up so I want to put a fresh brain effort into this.

Let ##A## be a nonempty subset of ##ℝ## which is bounded above.

We must show ##\exists x_n \in A \space | \space \lim_{n→∞} x_n = sup(A)##

We must somehow construct ##x_n## from the elements of A so that it converges to sup(A). Now because ##x_n \in A##, we know that ##x_n ≤ sup(A)##.

Using the hint given, I want to show the inequality ##x_n ≥ sup(A) - \frac{1}{n}## holds for any positive integer n because ##sup(A) - \frac{1}{n}## is not an upper bound for A. Inductively, take n=1. Then ##x_1 ≥ sup(A) - 1## and now assume this holds for any n. We want to show this holds for n+1, so ##x_{n+1} ≥ sup(A) - \frac{1}{n+1}##.

So we can inductively build our sequence ##x_n## with following the relation ##x_n ≥ sup(A) - \frac{1}{n}##.

Now notice that :
##x_n ≥ sup(A) - \frac{1}{n}##
##\Rightarrow sup(A) - x_n ≤ \frac{1}{n}##
##\Rightarrow |x_n - sup(A)| ≤ \frac{1}{n}##

So, ##\forall ε>0, \exists N \space | \space n ≥ \frac{1}{ε} \Rightarrow |x_n - sup(A)| < ε##

Thus making our choice of ##n≥\frac{1}{ε}## we observe ##|x_n - sup(A)| ≤ \frac{1}{n} < ε##.

Therefore we have found a way to construct a sequence of elements of A which converges to sup(A) as desired.

I know you said not to use induction micro, but I didn't seem to see any other way. Hopefully this makes sense.
 
  • #13
In the above, you haven't actually shown that [itex]x_n\in A[/itex]; you seem to be assuming this. Specifically, you need to show there exists a sequence that satisfies the inequality you want. Specifically, you need that for every positive integer, there exists an [itex]x_n \in A[/itex] such that [itex]sup(A)<x_n+1/n.[/itex] I would refer you to another problem you are working on that shows you that such an [itex]x_n[/itex] always exists..

ie. show that s=sup(A) if and only if a≤s for all a in A, and for all [itex]\epsilon>0[/itex], there exists [itex] a\in A[/itex] such that [itex]s-\epsilon<a[/itex].

Spoiler below (try working it out on your own first).

To get [itex]x_n=a[/itex], try taking [itex]1/n=\epsilon[/itex].
 
Last edited:
  • #14
christoff said:
In the above, you haven't actually shown that [itex]x_n\in A[/itex]; you seem to be assuming this. Specifically, you need to show there exists a sequence that satisfies the inequality you want. Specifically, you need that for every positive integer, there exists an [itex]x_n \in A[/itex] such that [itex]sup(A)<x_n+1/n.[/itex] I would refer you to another problem you are working on that shows you that such an [itex]x_n[/itex] always exists..

ie. show that s=sup(A) if and only if a≤s for all a in A, and for all [itex]\epsilon>0[/itex], there exists [itex] a\in A[/itex] such that [itex]s-\epsilon<a[/itex].

Spoiler below (try working it out on your own first).

To get [itex]x_n=a[/itex], try taking [itex]1/n=\epsilon[/itex].

Yes my wording was wrong there I meant to say because sup(A) exists, let me try this proof again.

Let ##A## be a nonempty subset of ##ℝ## which is bounded above.

We must show ##\exists x_n \in A \space | \space \lim_{n→∞} x_n = sup(A)##

We must somehow construct ##x_n## from the elements of A so that it converges to sup(A), but because sup(A) exists we know that ##x_n ≤ sup(A)##

So for any positive integer n, we must show ##sup(A) - \frac{1}{n} < x_n ≤ sup(A)##.

This comment made me blank for awhile :

ie. show that s=sup(A) if and only if a≤s for all a in A, and for all ϵ>0, there exists a∈A such that s−ϵ<a.

I don't see how that relates to this problem?
 
  • #15
Did you read my hint?

For n=1, you have [itex]\epsilon=1/1=1[/itex], so there exists an [itex]x\in A[/itex] such that
[itex]sup(A)-1<x[/itex]. Denote this x by [itex]x_1[/itex].

For n=2, you have [itex]\epsilon=1/2[/itex], so there exists an [itex]x\in A[/itex] such that [itex]sup(A)-1/2<x[/itex]. Now denote THIS x by [itex]x_2[/itex].

For n=3, [itex]\epsilon=1/3[/itex]. Construct [itex]x_3[/itex] the same way.

The existence of each [itex]x_n[/itex] at each step is guaranteed by the other problem I cited, which you just finished in another thread. In particular, such an [itex]x_n[/itex] exists for each n, and satisfies... which inequality?

As for why it's a sequence... to each n, you can associate an [itex]x_n[/itex], and each of these is in [itex]A[/itex]..
 
Last edited:
  • #16
christoff said:
Did you read my hint?

For n=1, you have [itex]\epsilon=1/1=1[/itex], so there exists an [itex]x\in A[/itex] such that
[itex]sup(A)-1<x[/itex]. Denote this x by [itex]x_1[/itex].

For n=2, you have [itex]\epsilon=1/2[/itex], so there exists an [itex]x\in A[/itex] such that [itex]sup(A)-1/2<x[/itex]. Now denote THIS x by [itex]x_2[/itex].

For n=3, [itex]\epsilon=1/3[/itex]. Construct [itex]x_3[/itex] the same way.

The existence of each [itex]x_n[/itex] at each step is guaranteed by the other problem I cited, which you just finished in another thread. In particular, such an [itex]x_n[/itex] exists for each n, and satisfies... which inequality?

As for why it's a sequence... to each n, you can associate an [itex]x_n[/itex], and each of these is in [itex]A[/itex]..

Ohh I see. Let me try again then.

Let ##A## be a nonempty subset of ##ℝ## which is bounded above.

We must show ##\exists x_n \in A \space | \space \lim_{n→∞} x_n = sup(A)##

So for any positive integer n, we must show ##sup(A) - \frac{1}{n} < x_n## so we can construct a sequence which is bounded by ##sup(A)##.

So take n=1, then we can find ##x_1 \in A## such that ##sup(A) - 1 < x_1## because sup(A)-1 is not an upper bound for A. We can follow this construction for any positive n yielding our desired inequality.

So we have successfully constructed our sequence ##x_n## by choosing the elements of A which satisfy ##sup(A) - \frac{1}{n} < x_n##. Now because sup(A) exists, we know that ##a ≤ sup(A), \space \forall a \in A##. Since each ##x_n \in A## for each positive integer n, we know that ##x_n ≤ sup(A)## for all n so that the sequence ##x_n## is bounded above by sup(A).

Hmm I don't have access to the monotone convergence theorem until literally one question after this. How would I finish this up without it? The same way I did before a few posts ago?
 
Last edited:
  • #17
Zondrina said:
Ohh I see. Let me try again then.

Let ##A## be a nonempty subset of ##ℝ## which is bounded above.

We must show ##\exists x_n \in A \space | \space \lim_{n→∞} x_n = sup(A)##

So for any positive integer n, we must show ##sup(A) - \frac{1}{n} < x_n##.

So take n=1, then we can find ##x_1 \in A## such that ##sup(A) - 1 < x_1## because sup(A)-1 is not an upper bound for A. We can follow this construction for any positive n yielding our desired inequality.

So we have successfully constructed our sequence ##x_n## by choosing the elements of A which satisfy ##sup(A) - \frac{1}{n} < x_n##. Now because sup(A) exists, we know that ##a ≤ sup(A), \space \forall a \in A##. Since each ##x_n \in A## for each positive integer n, we know that ##x_n ≤ sup(A)## for all n so that the sequence ##x_n## is bounded above by sup(A).

Hmm I don't have access to the monotone convergence theorem until literally one question after this. How would I finish this up without it? The same way I did before a few posts ago?

I think we are working on the same (or similar question). I just said since you have sup A -1/n < x ≤ sup A and then when you take the limit the left most inequality necessarily becomes a less or equal. So x is less or equal to sup A on both sides which means it is equal to sup A.
 
  • #18
CAF123 said:
I think we are working on the same (or similar question). I just said since you have sup A -1/n < x ≤ sup A and then when you take the limit the left most inequality necessarily becomes a less or equal. So x is less or equal to sup A on both sides which means it is equal to sup A.

Yes I was thinking about this, but I wasn't sure if I could take limits for granted. That would indeed show that ##lim(sup(A) - \frac{1}{n}) < lim(x_n) ≤ lim(sup(A))##, but because we can't have sup(A) < sup(A), it must be the case that ##lim(x_n) = sup(A)##.
 
  • #19
How to bypass this thing with the inequalities;
Note that if [itex]a<b[/itex], the we also have [itex]a\leq b[/itex]. So whenever you have a less-than sign, you can always instead write a less-than-or-equal-to sign.
Example: we have 1<2. Also, 1≤2.
On the other hand, the opposite is false; you can't necessarily turn a ≤ into a <.
Example: we have 2≤2. We don't have 2<2.

As for the aspect of "taking the limits for granted", here's a way to do it rigorously:

For all n>0, we have [itex]x_n-sup(A)\leq 0[/itex]. On the other hand, we have [itex]-\frac{1}{n}\leq x_n-sup(A)[/itex] for all n. Putting these two together, we have [itex]-\frac{1}{n}\leq x_n-sup(A)\leq 0 \leq \frac{1}{n}[/itex]. Hence, [itex]|x_n-\sup(A)|\leq\frac{1}{n}<\frac{2}{n}[/itex].

Now let [itex]\epsilon>0[/itex]. Then there exists an [itex]N>0[/itex] such that [itex]\frac{2}{N}<\epsilon[/itex] (just take N large enough; [itex]N>\frac{2}{\epsilon}[/itex]). Then for all [itex]n>N[/itex], we have [itex]|x_n-\sup(A)|<\frac{2}{n}<\frac{2}{N}=\epsilon[/itex]. Hence, [itex]x_n[/itex] converges to [itex]sup(A)[/itex] (by definition of convergence; epsilon definition).
 

Related to Can a Sequence of Elements in a Nonempty Subset Converge to Its Supremum?

1. What is a nonempty subset?

A nonempty subset is a set that contains at least one element. This means that the subset is not empty and has at least one object or element in it.

2. How do you determine if a subset is nonempty?

To determine if a subset is nonempty, you can check if it contains at least one element. If the subset has at least one element, then it is considered nonempty.

3. What is the difference between a nonempty subset and an empty subset?

A nonempty subset has at least one element, while an empty subset has no elements. In other words, a nonempty subset is not empty, while an empty subset is empty.

4. Why is it important to consider nonempty subsets?

Nonempty subsets are important because they allow us to define and analyze different properties of a set. For example, if a set is nonempty, then it has a smallest element, which can be useful in solving problems and proving theorems.

5. How do you prove that a subset is nonempty?

To prove that a subset is nonempty, you can either show that it contains at least one element, or you can use a proof by contradiction. In a proof by contradiction, you assume that the subset is empty and then show that this leads to a contradiction, which proves that the subset must be nonempty.

Similar threads

  • Calculus and Beyond Homework Help
Replies
11
Views
2K
  • Calculus and Beyond Homework Help
Replies
7
Views
1K
Replies
8
Views
3K
  • Calculus and Beyond Homework Help
Replies
15
Views
2K
  • Calculus and Beyond Homework Help
Replies
14
Views
2K
  • Calculus and Beyond Homework Help
Replies
3
Views
1K
  • Calculus and Beyond Homework Help
Replies
11
Views
2K
  • Calculus and Beyond Homework Help
Replies
6
Views
2K
  • Calculus and Beyond Homework Help
Replies
6
Views
2K
  • Calculus and Beyond Homework Help
Replies
14
Views
4K
Back
Top